What is the rate of change from x = 0 to x = pi over 2 ? (6 points) trig graph with points at (0, -4) and (pi over 2, 0) and (pi, 4) and (3 pi over 2, 0) and (2 pi, -4)

Answers

Answer 1

Answer: [tex]\dfrac{8}{\pi}[/tex] .

Step-by-step explanation:

We know that the rate of change of function f(x) from x=a to x= b is given by :-

[tex]k=\dfrac{f(b)-f(a)}{b-a}[/tex]

The given points on graph  :  (0, -4) and (pi over 2, 0) and (pi, 4) and (3 pi over 2, 0) and (2 pi, -4).

The rate of change from x = 0 to x = pi over 2 will be :-

[tex]\dfrac{0-(-4)}{\dfrac{\pi}{2}-0}=\dfrac{4}{\dfrac{\pi}{2}}[/tex]     [By using points (0, -4) and (pi over 2, 0) ]

[tex]=\dfrac{8}{\pi}[/tex]

Hence, the rate of change from x = 0 to x = pi over 2 is [tex]\dfrac{8}{\pi}[/tex] .


Related Questions

a Find the amount compounded annually on Rs 25,000 for 2 years if the rates of
interest for two years ore 10 % and 12 % respectively,​

Answers

Answer:

Amount = Rs. 30250 when Rate = 10%

Amount = Rs. 31360 when Rate = 12%

Step-by-step explanation:

Given

[tex]Principal, P = Rs.\ 25,000[/tex]

[tex]Time, t = 2\ years[/tex]

[tex]Rate; R_1 = 10\%[/tex]

[tex]Rate; R_2 = 12\%[/tex]

Number of times (n) = Annually

[tex]n = 1[/tex]

Required

Determine the Amount for both Rates

Amount (A) is calculated by:

[tex]A = P(1 + \frac{r}{n})^{nt}[/tex]

When Rate = 10%, we have:

Substitute 25,000 for P; 2 for t; 1 for n and 10% for r

[tex]A = 25000 * (1 + \frac{10\%}{1})^{1 * 2}[/tex]

[tex]A = 25000 * (1 + \frac{10\%}{1})^{2}[/tex]

[tex]A = 25000 * (1 + 10\%)^{2}[/tex]

Convert 10% to decimal

[tex]A = 25000 * (1 + 0.10)^{2}[/tex]

[tex]A = 25000 * (1.10)^{2}[/tex]

[tex]A = 25000 * 1.21[/tex]

[tex]A = 30250[/tex]

Hence;

Amount = Rs. 30250 when Rate = 10%

When Rate = 12%, we have:

Substitute 25,000 for P; 2 for t; 1 for n and 10% for r

[tex]A = 25000 * (1 + \frac{12\%}{1})^{1 * 2}[/tex]

[tex]A = 25000 * (1 + \frac{12\%}{1})^{2}[/tex]

[tex]A = 25000 * (1 + 12\%)^{2}[/tex]

Convert 12% to decimal

[tex]A = 25000 * (1 + 0.12)^{2}[/tex]

[tex]A = 25000 * (1.12)^{2}[/tex]

[tex]A = 25000 * 1.2544[/tex]

[tex]A = 31360[/tex]

Hence;

Amount = Rs. 31360 when Rate = 12%

Primo car rental agency charges $21 per day plus $0.20 por milo. Ultimo car rental agency charges $24 per day plus $1.00 per milo. Find the daily mileage for which the Ultimo charge is four times the Primo charge.
The mileage is

Answers

Answer:

300 miles

Step-by-step explanation:

Let us consider the miles they travelled is 'm'

Mileage for Primo= 21 + (m × 0.20) = 21+0.2m

Mileage for Ultimo= 24+ ( m× 1.00) = 24 + m

Question says The mileage is equal when Ultimo's charge is 4× Primo

Thus,

4 × (21+0.2m) = 24+ m

84 + 0.8m = 24 + m

60 = 0.2m

m = 300

If you randomly select a letter from the phrase "Sean wants to eat at Olive Garden," what is the probability that a vowel is randomly selected

Answers

Answer:

12/27

Step-by-step explanation:

Count all letters and all vowels then divide vowels by letters

The probability that a vowel is randomly selected in the experiment of selecting a letter from the phrase "Sean wants to eat at Olive Garden", is 4/9.

What is the probability of an event in an experiment?

The probability of any event suppose A, in an experiment is given as:

P(A) = n/S,

where P(A) is the probability of event A, n is the number of favorable outcomes to event A in the experiment, and S is the total number of outcomes in the experiment.

How to solve the given question?

In the question, we are given an experiment of selecting a letter from the phrase "Sean wants to eat at Olive Garden".

We are asked to find the probability that the selected letter is a vowel.

Let the event of selecting a vowel from the experiment of selecting a letter from the phrase "Sean wants to eat at Olive Garden" be A.

We can calculate the probability of event A by the formula:

P(A) = n/S,

where P(A) is the probability of event A, n is the number of favorable outcomes to event A in the experiment, and S is the total number of outcomes in the experiment.

The number of outcomes favorable to event A (n) = 12 (Number of vowels in the phrase)

The total number of outcomes in the experiment (S) = 27 (Number of letters in the phrase).

Now, we can find the probability of event A as:

P(A) = 12/27 = 4/9

∴ The probability that a vowel is randomly selected in the experiment of selecting a letter from the phrase "Sean wants to eat at Olive Garden", is 4/9.

Learn more about the probability of an event at

https://brainly.com/question/7965468

#SPJ2

An engineer wishes to determine the width of a particular electronic component. If she knows that the standard deviation is 3.6 mm, how many of these components should she consider to be 90% sure of knowing the mean will be within ± 0.1 ±0.1 mm?

Answers

Answer:

She must consider 3507 components to be 90% sure of knowing the mean will be within ± 0.1 mm.

Step-by-step explanation:

We are given that an engineer wishes to determine the width of a particular electronic component. If she knows that the standard deviation is 3.6 mm.

And she considers to be 90% sure of knowing the mean will be within ±0.1 mm.

As we know that the margin of error is given by the following formula;

The margin of error =  [tex]Z_(_\frac{\alpha}{2}_) \times \frac{\sigma}{\sqrt{n} }[/tex]  

Here, [tex]\sigma[/tex] = standard deviation = 3.6 mm

         n = sample size of components

         [tex]\alpha[/tex] = level of significance = 1 - 0.90 = 0.10 or 10%

         [tex]\frac{\alpha}{2} = \frac{0.10}{2}[/tex] = 0.05 or 5%

Now, the critical value of z at a 5% level of significance in the z table is given to us as 1.645.

So, the margin of error =  [tex]Z_(_\frac{\alpha}{2}_) \times \frac{\sigma}{\sqrt{n} }[/tex]  

                  0.1 mm        =  [tex]1.645 \times \frac{3.6}{\sqrt{n} }[/tex]

                    [tex]\sqrt{n} = \frac{3.6\times 1.645}{0.1 }[/tex]

                    [tex]\sqrt{n}[/tex] = 59.22

                     n = [tex]59.22^{2}[/tex] = 3507.0084 ≈ 3507.

Hence, she must consider 3507 components to be 90% sure of knowing the mean will be within ± 0.1 mm.

Help ive been stuck on this forever

Answers

Answer:

3480

Step-by-step explanation:

It always help if you make clear how the state assesses taxes. Usually it is done the way I will do it, but it is no guarantee.

First 3000 = 3000 * 2/100 = 60.00

The next 2000 (excess over 3000) = 2000 * 3/100 = 60

The next 12000 (excess over 5000) = 12000 * 5/100 = 600

The next step would be excess over 12000 = 48000 * 5.75/100 = 2760

That's the way most taxes for federal taxes and state taxes work. What you have to know is this: does the question use the term excess anywhere? Or do your notes.  If you are accustomed to the word then this is the way the question is done.

So the total taxes = 60 + 60 + 600 + 2760 = 3480

I need to find the distance B in the special counter sink shown

Answers

Answer:

Step-by-step explanation:

87°32' = 86°92'

(86°92')/2 = 43°46'

B = 13/(16cos(43°46')) = 1.125

Answer:

Step-by-step explanation:

Explain how to perform a​ two-sample z-test for the difference between two population means using independent samples with known.

Answers

Answer:

The steps 1-7 have been explained

Step-by-step explanation:

The steps are;

1) We will verify that the population standard deviations are known and that the population is normally distributed which means the sample size must be a minimum of 30.

2) We will state the null and alternative hypothesis

3) We will determine the critical values from the relevant tables

4) From the critical values gotten, we will determine it's corresponding region where it can be rejected.

5)We will calculate the value of the test statistic from the formula;

z = [(x1' - x2') - (μ1 - μ2)]/√[((σ1)²/n1) + ((σ2)²/n2)]

6) If the value of the test statistic gotten from step 5 above falls in the region of rejection noted in step 4,then we will reject the null hypothesis

7) After rejection of the null hypothesis, we will now give a decision/conclusion on the claim.

What is the solution to 4x + 2 = 6(-2x - 5) ?
O2
O 16
0-2
O -16
Please need help on this

Answers

x= -2
............
...........
&/&/&:&/&/&

4x+2=6(-2x-5)

<=>4x+2=-12x-30

<=>16x+32=0

<=>x=-2

two identical rubber balls are dropped from different heights. Ball 1 is dropped from a height of 109 feet, and ball 2 is dropped from a height of 260 feet. Use the function f(t) -16t^2+h to determine the current height, f(t), of a ball from a height h, over given time t.

When does ball 1 reach the ground? Round to the nearest hundredth​

Answers

Answer:  5.22 seconds

Step-by-step explanation:

t represents time and y represents the height.

Since we want to know when the ball hits the ground, find t when y = 0

Ball 1 starts at a height of 109 --> h = 109

0 = -16t² + 109

16t² = 109

   [tex]t^2=\dfrac{109}{16}\\[/tex]

   [tex]t=\sqrt{\dfrac{109}{16}}[/tex]

   [tex]t=\dfrac{\sqrt{109}}{2}[/tex]

   t = 5.22

Answer:Let us assume "H" height here, "t" as time.

=> H = 109

=> 0 = -16t² + 109

=> 16t² = 109

=> t² = 109/16

=> t = 109/2

=> t = 5.22 sec

Therefore, 5.22 second is the answer.

Determine which is the appropriate approach for conducting a hypothesis test. ​Claim: The mean RDA of sodium is 2400mg. Sample​ data: n​150, ​3400, s550. The sample data appear to come from a normally distributed population.

Answers

Answer:

Use the student t distribution

Step-by-step explanation:

Here is the formula

t = (x - u) ÷(s/√N)

From the information we have in the question:

n = 150

s = 550

x = 3400

u = mean = 2400

= 3400 - 2400÷ 500/√150

= 1000/44.9

= 22.27

At 0.05 significance level, df = 149 so t tabulated will be 1.65.

We cannot use normal distribution since we do not have population standard deviationWe cannot use normal distribution since we do not have population standard deviationChisquare cannot be used since we are not testing for population varianceWe cannot use normal distribution since we do not have population standard deviationChisquare cannot be used since we are not testing for population varianceThe parametric or bootstrap method cannot be used either.

An oil company is going to issue new ID codes to its employees. Each code will have one letter, followed by one digit, followed by three letters. The letters w, x, y, and z will not be used. So, there are 22 letters and 10 digits that will be used. Assume that the letters can be repeated. How many employee ID codes can be generated?

Answers

Answer:

2342560 combos

Step-by-step explanation:

so its 1 letter*1number*1 letter*1 letter*1 letter, or 22x10x22x22x22 which should equate to 2342560 possible ID codes, hope this helps :)

If mowing burns average $115 over 20 minutes how many calories are you burning in one hour

Answers

Answer:

345

Step-by-step explanation:

20*3 = 60 there's 60 minutes in one hour

115*3 = 345

Find the length of side

x in simplest radical form with a rational denominator.



Thanks in advance

Answers

Answer:

2

Step-by-step explanation:

Pythagoras. c² = a² + b²

since both "side angles" are equal (45 degrees), we know it is an isosceles triangle, that means also the other side = x.

and so,

8 = x² + x² = 2x²

4 = x²

x = 2

Answer:

x = 2

Step-by-step explanation:

sin(45)/x = sin(90)/[tex]\sqrt{8}[/tex]

[tex]\sin \left(45^{\circ \:}\right)=\frac{\sqrt{2}}{2}[/tex]

x = [tex]\sqrt{8}[/tex] [tex]\sin \left(45^{\circ \:}\right)[/tex]

[tex]x = \sqrt{8} \frac{\sqrt{2}}{2}[/tex]

x = [tex]\frac{\sqrt{16} }{2}[/tex]

x = 4/2

x = 2

Please answer quick!!!

Find the interquartile range of the data set represented by this box plot.

30
56
20
10

Answers

Answer:

A. 30

Step-by-step explanation:

The interquartile range for a box and whiskers plot, is the value from the right side of the box minus the value of the left side of the box.

In this case at the far right side of the box it is at 130, at the far left side of the box it is at 100.

130-100=30

Answer:

[tex]\huge\boxed{IQR = 30}[/tex]

Step-by-step explanation:

Q1 = 130 (Left hand edge of the box)

Q3 = 100 (Right Hand edge of the box)

Interquartile Range = Q3-Q1

IQR = 130-100

IQR = 30

Find the least common multiple of 14 and 22.

Answers

Answer:

154

Step-by-step explanation:

14: 14, 28, 42, 56, 70, 84, 98, 112, 126, 140, 154,…

22: 22, 44, 66, 88, 110, 132, 154,…

The least common multiple is 154
The right answer is 154 hope this helped

Please help. I’ll mark you as brainliest if correct!

Answers

Answer:

x and y can have many values

Step-by-step explanation:

-24x - 12y = -16

Then: 24x + 12y = 16

We know: 6x + 3y = 4

X and Y can have a lot of valoues.

6x + 3y = 4

3 ( 2x + y) = 4

2x + y= 4/3

2x+y= 1.333...

What is the volume of a square pyramid whose length of one side of its base is 9cm and whose height is 15cm. Show your work

Answers

Answer:

The answer is 405cm³

Step-by-step explanation:

Volume of a pyramid is given by

[tex]V = \frac{1}{3} \times area \: of \: base \: \: \times height[/tex]

height = 15cm

From the question the pyramid is a square pyramid which means it's base is a square

Area of a square = l²

where l is the length of one side

l = 9cm

Area of square = 9² = 81cm²

So the volume of the pyramid is

[tex]V = \frac{1}{3} \times 81 \times 15[/tex]

[tex]V = 27 \times 15[/tex]

We have the final answer as

V = 405 cm³

Therefore the volume of the pyramid is

405cm³

Hope this helps you

If a= -5 and b= -2,then what is the value of [a-b]2

Answers

substitute= -5–2 x 2
solve= 10x2
=20

Step-by-step explanation:

Substitute the values of a and b into [a-b]2

= [-5-(-2)]2

= [-5+2]2

= [-3]2

= -6

find the measure of the angle indicated

Answers

Answer:

Step-by-step explanation:

59°

Calculating the degrees of freedom, the sample variance, and the estimated standard error for evaluations.

using the t statistic.

With another study, where you also plan on evaluating a mean using the t statistic, you have a sample of n = 21 that has an SS of 500. What is the variance for the sample?

A. 5.00

B. 22. 36

C. 25

D. 250,000

Answers

Answer:

The variance is  [tex]\sigma ^2 =25[/tex]

Step-by-step explanation:

From the question we are told that

    The sample size is  n =  21

     The sum of squares is [tex]SS = 500[/tex]

Generally the variance is mathematically represented as

           [tex]\sigma ^2 = \frac{SS}{n- 1}[/tex]

substituting values

          [tex]\sigma ^2 = \frac{ 500}{21- 1}[/tex]

          [tex]\sigma ^2 =25[/tex]

   

Given the function f ( x ) = 2 x + 8 , evaluate and simplify the expressions below. See special instructions on how to enter your answers.

Answers

Answer:

[tex]f(a) = 2a + 8[/tex]

[tex]f(x + h) = 2x + 2h + 8[/tex]

[tex]\frac{f(x + h) - f(x)}{h} = 2[/tex]

Step-by-step explanation:

Given

[tex]f(x) = 2x + 8[/tex]

Required

[tex]f(a)[/tex]

[tex]f(x + h)[/tex]

[tex]\frac{f(x + h) - f(x)}{h}[/tex]

Solving for f(a)

Substitute a for x in the given parameter

[tex]f(x) = 2x + 8[/tex] becomes

[tex]f(a) = 2a + 8[/tex]

Solving for f(x+h)

Substitute x + h for x in the given parameter

[tex]f(x + h) = 2(x + h) + 8[/tex]

Open Bracket

[tex]f(x + h) = 2x + 2h + 8[/tex]

Solving for [tex]\frac{f(x + h) - f(x)}{h}[/tex]

Substitute 2x + 2h + 8 for f(x + h), 2x + 8 fof f(x)

[tex]\frac{f(x + h) - f(x)}{h}[/tex] becomes

[tex]\frac{2x + 2h + 8 - (2x + 8)}{h}[/tex]

Open Bracket

[tex]\frac{2x + 2h + 8 - 2x - 8}{h}[/tex]

Collect Like Terms

[tex]\frac{2x - 2x+ 2h + 8 - 8}{h}[/tex]

Evaluate the numerator

[tex]\frac{2h}{h}[/tex]

[tex]2[/tex]

Hence;

[tex]\frac{f(x + h) - f(x)}{h} = 2[/tex]

The U.S. Dairy Industry wants to estimate the mean yearly milk consumption. A sample of 21 people reveals the mean yearly consumption to be 74 gallons with a standard deviation of 16 gallons. Assume that the population distribution is normal. (Use t Distribution Table.)
a-1. What is the value of the population mean?
16
Unknown
74
a-2. What is the best estimate of this value?
Estimate population mean
c. For a 90% confidence interval, what is the value of t? (Round your answer to 3 decimal places.)
Value of t
d. Develop the 90% confidence interval for the population mean. (Round your answers to 3 decimal places.)
Confidence interval for the population mean is and .
e. Would it be reasonable to conclude that the population mean is 68 gallons?
a) Yes
b) No
c) It is not possible to tell.

Answers

Correct question is;

The U.S. Dairy Industry wants to estimate the mean yearly milk consumption. A sample of 21 people reveals the mean yearly consumption to be 74 gallons with a standard deviation of 16 gallons.

a. What is the value of the population mean? What is the best estimate of this value?

b. Explain why we need to use the t distribution. What assumption do you need to make?

c. For a 90 percent confidence interval, what is the value of t?

d. Develop the 90 percent confidence interval for the population mean.

e. Would it be reasonable to conclude that the population mean is 68 gallons?

Answer:

A) Best estimate = 74 gallons

B) because the population standard deviation is unknown. The assumption we will make is that the population follows the normal distribution.

C) t = 1.725

D) 90% confidence interval for the population mean is (67.9772, 80.0228) gallons

E) Yes

Step-by-step explanation:

We are given;

Sample mean; x' = 74

Sample population; n = 21

Yearly Standard deviation; s = 16

A) We are not given the population mean.

So the closest estimate to the population mean would be the sample mean which is 74.

B) We are not given the population standard deviation and as such we can't use normal distribution. So what is used when population standard deviation is not known is called t - distribution table. The assumption we will make is that the population follows the normal distribution.

C) At confidence interval of 90% and DF = n - 1 = 21 - 1 = 20

From t-tables, the t = 1.725

D) Formula for the confidence interval is;

x' ± t(s/√n) = 74 ± 1.725(16/√21) = 74 ± 6.0228 = 67.9772 or 80.0228

Thus 90% confidence interval for the population mean is (67.9772, 80.0228) gallons

E) 68 gallons lies within the range of the confidence interval, thus we can say that "Yes, it is reasonable"

London bought snacks for her team's practice. She bought a bag of apples for $2.25
and a 18-pack of juice bottles. The total cost before tax was $9.63. Write and solve an
equation which can be used to determine j, how much each bottle of juice costs?

Answers

Answer:

9.63 - 2.25 = 18j

j = 0.41

Step-by-step explanation:

first you set the equation equal to 18 since you want to find out what each bottle of juice  costs.

= 18j

if the total cost was 9.63 you need to subtract 2.25 form it to find out how much the 18-pack of juice bottles was. so you set 9.63 - 2.25 equal to  18j

9.63 - 2.25 = 18j

7.38 = 18j

0.41 = j

Check your work:

9.63 - 2.25 = 18(0.41)

7.38 = 7.38                    true!

hope this helps! if you have any questions, let me know!

A company finds that the rate at which the quantity of a product that consumers demand changes with respect to price is given by the​ marginal-demand function Upper D prime (x )equals negative StartFraction 5000 Over x squared EndFraction where x is the price per​ unit, in dollars. Find the demand function if it is known that 1006 units of the product are demanded by consumers when the price is ​$5 per unit.

Answers

Answer:

q =  5000/x  + 6

Step-by-step explanation:

D´= dq/dx  =  - 5000/x²

dq = -( 5000/x²)*dx

Integrating on both sides of the equation we get:

q = -5000*∫ 1/x²) *dx

q = 5000/x + K   in this equation x is the price per unit and q demanded quantity and K integration constant

If when  1006 units are demanded when the rice is 5 then

x = 5     and   q = 1006

1006  =  5000/5 +K

1006 - 1000 = K

K = 6

Then the demand function is:

q =  5000/x  + 6

Select the correct answer -1/4(12x+8) is less than it equal to -2x+11

Answers

Answer:

x ≤ [tex]\frac{9}{5}[/tex]

Step-by-step explanation:

Given

[tex]\frac{1}{4}[/tex](12x + 8) ≤ - 2x + 11 ← distribute parenthesis on left side

3x + 2 ≤ - 2x + 11 ( add 2x to both sides )

5x + 2 ≤ 11 ( subtract 2 from both sides )

5x ≤ 9 ( divide both sides by 5 )

x ≤ [tex]\frac{9}{5}[/tex]

Answer: x≤ 3/-10

Explanation: Firstly given that

-¼(12x+8) ≤ -2x+11

• Divide by 4

4X-¼(12x+8) ≤-2x+11

= -12x + 8 ≤ -2x + 11

• Group like terms

-12x + 2x ≤ 11 - 8

= -10x/10 ≤ 3/-10

x≤ 3/-10

What is the slope of the line that goes through the points (-2, 4) and (5, -1)

Answers

Answer:

  -5/7

Step-by-step explanation:

The slope of a line is given by

m = (y2-y1)/(x2-x1)

   = ( -1 -4)/(5 - -2)

   = (-1-4)/(5+2)

   -5/7

Slope formula: y2-y1/x2-x1

= -1-4/5-(-2)

= -5/7

Best of Luck!

Take thus quote, and embed (introduce) it into a complete sentence: "TV plots
and characters tended to be simple" The author is Ostergaard.

Answers

TV plots and characters tended to be simple, no more no less than expected.

Plzz help i cant figure this out..

Answers

Answer:

[tex]\large \boxed{\mathrm{B. \ \ \{-10, -6, 10\} }}[/tex]

Step-by-step explanation:

The domain is the x values.

D = {-1, 0, 4}

y = 4(-1) - 6 = -4 - 6 = -10

y = 4(0) - 6 = 0 - 6 = -6

y = 4(4) - 6 = 16 - 6 = 10

The range is the y values.

R = {-10, -6, 10}

pril Heights (in inches) June Heights (in inches) 15 23 11 42 45 44 39 19 20 12 45 45 40 43 14 12 13 41 40 45 41 the difference between the mean height in April and the mean height in June? 34 inches B. 33 inches C. 30 inches D. 28 inches​

Answers

Answer:

I need points plsss

Step-by-step explanation:

During the 2014 season, the Los Angeles Dodgers won 58% of their games. Assuming that the outcomes of the baseball games are independent and that the percentage of wins this season will be the same as in 2014: What is the probability that the Dodgers will win at least one of their next seven games

Answers

Answer: 0.98

Step-by-step explanation:

given data:

probability they won a game = 58% = 0.58

since outcome of games are independent, and percentage would remain same as 2014.

probablility that Dodgers wins atleast 1 of their next 7 games

= 1 - p

= 1 - ( 0.58 )^ 7

= 1 - 0.02208

= 0.98

probabikotun that Dodgers would win one of their next seven games is 0.98

Other Questions
a college entrance exam company determined that a score of 25 on the mathematics portion of the exam suggests that a student is ready for Benjamin Franklin said, "An ounce of prevention is worth a pound of cure." This can be applied to your physical health, mental health, and fitness. What do you think this means? Need help finding the value for A On the first two downs, a football team lost a yard and then lost 2 yards.Write an expression with addition that shows the changes in yards onthe first two downs. Hannahs mom bought a variety pack of chips. There were a total of 12 bags. Her mom put the bags in a jar. There were 5 barbecue,3 plain,3 sour cream, and 1 tortilla.What is the probability of Hannah selecting a bag of barbecue chips if she doesnt look in the jar?A.) 5/12B.) 5/7C.) 12/5D.) 1/12E.) 7/12 Explain who the Spanish conquistadores are/were (quick summary would be extremely helpful!) WHICH PROGRAMMING LANGUAGES ARE THE BEST AND COMPATIBLE FOR 3D PRINTERS? When the input is 1, what is the output of the function? That is, find the value of g(1). How did the plantation system affect life for African slaves living in Europeancolonies in the Americas?O A. It led to high death rates for African slaves who were forced towork under brutal conditions.O B. It allowed African slaves to wield political authority by ruling overindigenous laborers.O C. It convinced African slaves to convert to Catholicism in order toreceive political freedom.OD. It forced African slaves to regularly travel between Africa and theAmerican colonies. Draw all four products when the following compound undergoes dehydrohalogenation and rank them in terms of stability. Which product do you expect to be the major product? help me with these thanks What is the measure of the unknown angle?Image of a straight angle divided into two angles. One angle is thirty five degrees and the other is unknown. high priority should be given for the development and use oc hydroelectricity justify with reason Find all solutions to the equationin the interval [0, 21). Enter thesolutions in increasing order. Each lap around a park is 1 15 miles. Kellyn plans to jog at least 7 12 miles at the park without doing partial laps. How many laps must Kellyn jog to meet her goal? Three causes and effects of self image Can you please help me on add and subtract fractions level 1 9/8-11/8 Please answer this question now Match each leadership skill to its advantage.communicationgoal settingchange managementteam buildinghelps the group adapt to new developmentshelps individuals work togetherwhile supporting each otherhelps individuals measure their progresshelps the leader clarify team understandings when an apple falls towards the earth the earth moves up to meet the apple. Is it true? If yes, why is the earth motion not noticeable ?